Anda di halaman 1dari 7

EMGT 346 HOMEWORK #1 SOLUTIONS

1-2
(a) Yes. The choice of an engine has important money consequences, so it
would be suitable for engineering economic analysis.
(b) Yes. Important economic and social consequences. Some might argue that the
social consequences are more important than the economics.
(c) ? Probably there are a variety of considerations much more important than the
economics.
(d) No. Picking a career on an economic basis sounds terrible.
(e) No. Picking a wife on an economic basis sounds even worse.

1-18
(a) Maximize the difference between output and input.
(b) Minimize input.
(c) Maximize the difference between output and input.
(d) Minimize input.

1-25
In decision making the model is mathematical.

1-26
The situation is an example of the failure of a low-cost item that may have major
consequences in a production situation. While there are alternatives available, one
appears so obvious that that foreman discarded the rest and asks to proceed with
the replacement.
One could argue that the foreman, or the plant manager, or both are making
decisions. There is no single right answer to this problem.

1-59
Saving = 2 [$185.00 + (2 90 miles) ($0.60/mile)] = $586.00/week

2-2
(a) 500 parts
Average cost = $13
Marginal cost = $13
(b) 1500 parts
Average cost = ((1000)($13) + (500)($12)) / 1500 = $ 12.67
Marginal cost = $12
(c) 2500 parts
Average cost = ((1000)($13) + (1500)($12)) / 2500 = $12.40
Marginal cost = $12
(d) 3500 parts
Average cost = ((1000)($13) + (2000)($12) + 500($11)) / 3500 = $12.14
Marginal cost = $11

2-6
x = no. of maps dispensed per year
(a) Fixed Cost (I) = $1,000
(b) Fixed Cost (II) = $5,000
(c) Variable Costs (I) = 0.900
(d) Variable Costs (II) = 0.100
(e) Set Total Cost (I) = Total Cost (II)
$1,000 + 0.90 x = $5,000 + 0.10 x
thus x = 5,000 maps dispensed per year.
The student can visually verify this from the figure.
(f) System I is recommended if the annual need for maps is <5,000
(g) System II is recommended if the annual need for maps is >5,000
(h) Average Cost @ 3,000 maps:
TC(I) = (0.9) (3.0) + 1.0 = 3.7/3.0 = $1.23 per map
TC(II) = (0.1) (3.0) + 5.0 = 5.3/3.0 = $1.77 per map
Marginal Cost is the variable cost for each alternative, thus:
Marginal Cost (I) = $0.90 per map
Marginal Cost (II) = $0.10 per map

2-12
x = units/year
By hand = Painting Machine
$1.40 x = $15,000 + $0.20x
x = $15,000/1.20 = 12,500 units
12,500 / 3 = 4,167 units

2-22
This is an example of a sunk cost. The $7,000 is a past cost and should not be
allowed to alter a subsequent decision unless there is some real or perceived effect.
Since either home is really an individual plan selected by the homeowner, each
should be judged in terms of value to the homeowner vs. the cost. On this basis the
stock plan house appears to be the preferred alternative.

2-36
(a) Unit Profit = $197 (0.30)= $59.10 or
= Unit Sales Price Unit Cost
= $197 (1.3) $197 = $256.10 $197 = $59.10
(b) Overall Batch Cost = $197 (10,000) = $1,970,000
(c) Of the 10,000 batch:
1. (10,000) (0.01) = 100 are scrapped in mfg.
2. (10,000 100) (0.03) = 297 of finished product go unsold
3. (9,900 297) (0.02) = 192 of sold product are not returned
Total = 589 of original batch are not sold for profit
Overall Batch Profit = (10,000 589) $59.10 = $556,190.10
(d) Unit Cost = 63 ($0.50) + $24 + $110 = $165.50
Batch Cost with Contract = 10,000 (165.50) = $1,655,000

Difference in Batch Cost:


= BC without contract- BC with contract = $1,970,000 $1,655,000
= $315,000
SungSam can afford to pay up to $315,000 for the contract.

2-41
Equipment
Varnish Bath
Power Scraper
Paint Booth

Cost of New Equipment minus


(75/50)0.80 (3,500) = $4,841
(1.5/0.75)0.22 (250) = $291
(12/3)0.6 (3,000) = $6,892

Trade-In Value
$3,500 (0.15)
$250 (0.15)
$3,000 (0.15)
Total

2-49
T (25) = 0.60 (25log (0.75)/log (2.0)) = 0.16 hours/unit
Labor Cost = ($20/hr) (0.16 hr/unit) = $3.20/unit
Material Cost = ($43.75/25 units) = $1.75/unit
Overhead Cost = (0.50) ($3.20/units) = $1.60/unit
Total Mfg. Cost = $6.55/unit
Profit = (0.20) ($6.55/unit) = $1.31/unit
Unit Selling Price = $7.86/unit

2-53
Year
0.00
1.00
2.00
3.00
4.00
5.00
6.00
7.00

Capital Costs
20
0
0
0
0
0
0
2

O&M
0
2.5
2.5
2.5
2.5
2.5
2.5
2.5

Overhaul
0
0
0
0
5
0
0
0

3-2
$2,000 + $2,000 (0.10 3) = $2,600

3-16
F = P (1 + i)n
Solve for P: P = F/(1 + i)n
P = F (1 + i)n
P = $150,000 (1 + 0.10)5 = $150,000 (0.6209) = $93,135

3-18
Use F = P (F/P, i, n) = P (1 + i)n = 2000 (1 + 0.06)n.
(a) n = 5, F = $2,676
(b) n = 10, F = $3,582
(c) n = 20, F = $6,414
(d) n = 50, F = $36,840

= Net Cost
= $4,316
= $254
= $6,442
$11,012

(e) n = 100, F = $678,604

3-22
Use F = P (F/P, 8%, n) = 1000 (1 + 0.08)n .
(a) F = 1360, n = log(1.36)/log(1.08) = 4 years
(b) F = 2720, n = log(2.72)/log(1.08) = 13 years
(c) F = 4316, n = log(4.316)/log(1.08) = 19 years
(d) F = 6848, n = log(6.848)/log(1.08) = 25 years

3-31
Calculator Solution
1% per month F = $1,000 (1 + 0.01)12 = $1,126.83
12% per year F = $1,000 (1 + 0.12)1 = $1,120.00
Savings in interest = $6.83
Compound interest table solution
1% per month F = $1,000 (1.127) = $1,127.00
12% per year F = $1,000 (1.120) = $1,120.00
Savings in interest = $7.00

3-37
Either:
Q10 = Q6 (F/P, 5%, 4) (1)
Q10 = P (F/P, 5%, 10) (2)
Since P is between and Q6 is not, solve Equation (2),
Q10 = $60 (1.629)
= $97.74

3-39
Effective Interest Rate = (1 + 0.0175)12 1 = 0.2314 = 23.14%

3-40
Effective Interest Rate = (1 + i)m 1 = (1.03)4 1 = 0.1255 = 12.55%

3-46
P = $9,500, F = $10,000, i = ?, n = 1 six-month interest period
F = P (1 + i)
(1 + i) = F/P = $10,000/$9,500 = 1.0526
i = .0526 = 5.26%
Nominal Interest Rate = 5.26% (2) = 10.52%
Effective Interest Rate = (1 + .0526)2 1 = 0.10797 = 10.80%

4-2
F = $100 (F/A, 10%, 3)
= $100 (3.310)
= $331
P = $331 (F/P, 10%, 2) = $331 (1.210)
= $400.51
J = $400.51 (A/P, 10%, 3) = $400.51 (0.4021) = $161.05
Alternate Solution:
One may observe that J is equivalent to the future worth of $100 after five interest
periods, or:
J = $100 (F/P, 10%, 5) = $100 (1.611) = $161.10

4-7
P = $3,000 + $480 (P/A, 1%, 60) = $3,000 + $480 (44.955)
= $21,578.40

4-8
F = $2,000(F/A, 8%, 10)(F/P, 8%, 5)
= $2,000 (14.487) (1.469)
= $42,560

4-46
FW = FW
$1000 (F/A, i%, 10) (F/P, i%, 4) = $28,000
By trial and error:
Try i = 12% $1,000 (17.549) (1.574) = $27,622 i too low
i = 15% $1,000 (20.304) (1.749) = $35,512 i too high
Using Interpolation:
i = 12% + 3% (($28,000 $27,622)/($35,512 $27,622))
= 12.14%

4-60
Present worth of gradient series:
P = $100 (P/G, 10%, 4) = $100 (4.378) = $437.80
D = $437.80 (A/F, 10%, 4) = $437.80 (0.2155) = $94.35

4-71
Assuming end of year payment for maintenance costs:
P' = $80 (P/A, 8%, 4) + $40 (P/G, 8%, 4)
P' = $80 (3.312) + $40 (4.650) = 264.96 + 186 = 450.96
P = P' (P/F, 8%, 2) = $450.96 (0.8573) = $386.61

4-83

(a) P = 50,000 (P/A, 7%, 15) + 5,000 (P/G, 7%, 15)


= 50,000 (9.108) + (5,000) (52.446)
= $717,630
(b) P = 50,000 (P/A, 10%, 7%, 15)
= 50,000

1 (1+0.10)15 (1+0.07)-15
--------------------------------------------0.07 0.10

= $856,712

4-90
(a) Since the book only gives a geometric gradient to present worth factor, we must
first solve for P and then F.
P = ? n = 6 i = 10%
g = 8%
P = A1(P/A, g%, i%, n)
(P/A, g%, i%, n) = [(1 (1 + g)n (1 + i)n)/(i g)]
= [(1 (1.08)6 (1.10)6)/(0.10 0.08)]
= 5.212
P = $1,500 (5.212) = $7,818
F = P (F/P, i%, n) = $7,818 (F/P, 10%, 6) = $13,853
As a check, solve with single payment factors:
$1,500.00 (F/P, 10%, 5) = $1500.00 (1.611) = $2,413.50
$1,620.00 (F/P, 10%, 4) = $1,620.00 (1.464) = $2,371.68
$1,749.60 (F/P, 10%, 3) = $1,749.60 (1.331) = $2,328.72
$1,889.57 (F/P, 10%, 2) = $1,898.57 (1.210) = $2,286.38
$2,040.73 (F/P, 10%, 1) = $2,040.73 (1.100) = $2,244.80
$2,203.99 (F/P, 10%, 0) = $2,203.99 (1.000) = $2,203.99
Total Amount = $13,852.07
(b) Here, i% = g%, hence the geometric gradient to present worth equation is
P = A1 n (1 + i)1 = $1,500 (6) (1.08)1 = $8,333
F = P (F/P, 8%, 6) = $8,333 (1.587) = $13,224

4-99
(a) A = P (A/P, 1.5%, 60) = $635
(b) $635 = $24,500 (A/P, X, 60)
635 / 24500 = 0.0259 ~ 1.54%
ia = (1 + 0.0154)12 1 = 20.13%

4-101
(a) r = I x m = (1.25%) (12) = 15%
(b) ia = (1 + 0.0125)12 1 = 16.08%

A = $10,000 (A/P, 1.25%, 48) = $10,000 (0.0278) = $278

4-111
Compute the effective interest rate per quarterly payment period:
iqtr = (1 + 0.10/12)3 1 = 0.0252 = 2.52%
Compute the present worth of the 32 quarterly payments:
P = A (P/A, 2.52%, 32)
= $3,000 [(1.0252)32 1]/[0.0252(1.0252)32]
= $3,000 (21.7878)
= $65,363

Anda mungkin juga menyukai